If x=3, y=-2, z=-1 ,find the value of 3x-4y+z
I really need help

Answers

Answer 1

Answer:

16

Step-by-step explanation:

Since, x=3, y=-2, z=-1

We can Substitute:

3(3)-4(-2) + -1

9 -4(-2)+-1

9 + 8 + -1

17 - 1

16

~Lenvy~


Related Questions

What points should I circle and in which point do they intersect?

I WILL MARK BRAINLIEST AND GIVE THANKS

Answers

Answer:

[tex]\displaystyle [3, -2][/tex]

Explanation:

All you do is substitute three in for [tex]\displaystyle x[/tex]to get the y-coordinate of [tex]\displaystyle -2.[/tex]This means that these equations meet at the ordered pair of [tex]\displaystyle [3, -2].[/tex]

I am joyous to assist you at any time.

Volt Electronics sells a Y-Box game console for $207. The store markup on thr Y-Box is 38%. What was the wholesale price that the store paid for it?​

Answers

The wholesale price that the store paid for the Y-Box game is $150.

What is the wholesale price?

When the price of an item is marked up, it means that the price of the item was increased by a certain percentage. The good then becomes more expensive.

Wholesale price = selling price / ( 1 + percentage mark up)

$207 / (1+ 38)

$207 / (1.38) = $150

To learn more about profit, please check: https://brainly.com/question/26181966

PLEASE HELP ME! THERE IS A PICTURE

Answers

B. 1/40…. You divide 1/2 by 20 to find out how much hand sanitizer was used on each desk. You can keep change flip meaning keep the 1/2 change the division to multiplication and flip the 20 to 1/20 then multiply. 1/2 by 1/20 which equals 1/40.

Answer:

I think 10

letter C.

Step-by-step explanation:

divide *2

Prism M is a dilation of Prism N. The height of Prism M is 4 3/4 ft, and the volume of Prism M is 68 2/5 ft³. The height of Prism N is 2 3/8 ft. What is the volume of Prism N?Enter your answer as a mixed number in simplest form by filling in the boxes.

Answers

Answer:

  8 11/20 ft³

Step-by-step explanation:

For dilated figures, the ratio of volumes is the cube of the ratio of corresponding linear dimensions.

  Vn/Vm = (Hn/Hm)³

  Vn = Vm(Hn/Hm)³ = (68 2/5 ft³)((2 3/8 ft)/(4 3/4 ft))³

  Vn = (342/5 ft³)((19/8)/(19/4))³ = (342/5 ft³)(1/2)³ = 342/40 ft³

  Vn = 8 11/20 ft³

The volume of Prism N is 8 11/20 ft³.

Answer:

The answer is 8 11/20.

The first term of a geometric sequence is 4 and grows exponentially by a factor of 3. Murphy writes out the terms and says that the sum of the 4th and 5th terms is​ 1,296. Explain​ Murphy's error and correct it. (Fill in the blanks)
Murphy added terms
____
together. The correct sum of the 4th and 5th terms is ___
​(Simplify your​ answer.)

Answers

The geometric sequence can increase or decrease exponentially

The correct sum of the 4th and the 5th terms is 432

How to determine the correct sum?

The given parameters are:

Initial value (a) = 4

Growth factor (r) = 3

The nth term of a geometric sequence is calculated as:

[tex]T_n = ar^{n-1}[/tex]

This gives

[tex]T_n = 4 * 3^{n-1}[/tex]

For the 4th term; we have:

[tex]T_4 = 4 * 3^{4-1}[/tex]

[tex]T_4 = 108[/tex]

For the 5th term, we have:

[tex]T_5 = 4 * 3^{5-1}[/tex]

[tex]T_5 = 324[/tex]

Add the 4th and the 5th terms

Sum = 108 + 324

Sum = 432

Hence, the correct sum of the 4th and the 5th terms is 432

Read more about geometric sequence at:

https://brainly.com/question/24643676

In the figure below, mR is 66°, and mT is 130°. Note: Figure is not drawn to scale. What is mQ? A. 24° B. 50° C. 64° D. 116°

Answers

Answer:

b

Step-by-step explanation:

i just think so correct me if im wrong

Use the dot plot to answer the question.

A dot plot that has 3 dots at 1, 4 dots at 2, 3 dots at 3, 4 dots at 4, 5 dots at 5, & 3 dots at 6.

What is the mean of the data set in the dot plot?

Enter your answer as a number rounded to the nearest tenth, like this: 42.5

Answers

The mean of the data set is 2.20

Factor: 35b + 15c -10

Answers

Answer:

5(7b+3c-2)

Step-by-step explanation:

Common factor of 5

35b/5= 7b
15c/5= 3c
-10/5= -2

Take the common factor and multiply it by the quotients
5(7b+3c-2)

Use your TI-83 to determine the necessary t* value for a 90% confidence interval
based on a sample size of 55 and a sample standard deviation of 7.42.

2.005
1.669
1.96
1.645
2.67

Answers

Answer:

Step-by-step explanation:

It is 1.645. My calculator is a little off though; see attached picture.

can you pls help me i need it?

Answers

Answer:

$114.75

Step-by-step explanation:

You have to multiply the hours by the wage. Looking at the time that the worker was in during the morning, it was a total of 4 hours. Since the wage is $13.50/hr, we would multiply 13.50 by 4.

13.50 * 4 = 54.00

So, now we have to add together the total hours in the afternoon. If we count the time, we get 4 1/2 hours. So, now we multiply 13.50 by 4.5.

13.50 * 4.5 = 60.75

Now, to find the total pay for that day, we add both the morning and the afternoon pay together.

54.00 + 60.75 = 114.75

Therefore, the pay for this day is $114.75.

Question 1-5
Sarah is demonstraing the idea that polynomials are closed under addition. Her work and explanation are shown.
Select an equation and phrases to complete the statement.
Part A:
Add 3x4 + 6x - 7 to
Equation A 2x - (2x)2 + 4x-1
Equation B (2x)2 + 4.4x - 9
Equation C (-8x)-2 - (3x)3 +7
Part B:
This is closed because
of the sum are
and the sum is a polynomial.

Answers

This is closed under addition because the sum of the polynomial produces another polynomial.

What is a polynomial?

A polynomial is an expression that involves the addition, subtraction and multiplication of variables.

Polynomials will be closed under an operation if the operation produces another polynomial.

(3x⁴ + 5x - 7) + (2x - (2x)² + 4x - 1) + [(-8x)⁻² - (3x)³ + 7]

= 3x⁴ - 27x³ - 4x² + 9x - 1 + (-8x)⁻²

This is closed under addition because the sum of the polynomial produces another polynomial.

Find out more on polynomial at: https://brainly.com/question/2833285

hello, me again polease help me with this, and show work

Answers

Answer:

Hope it helps u

Step-by-step explanation:

1.7

1.749

1.75

1.775

1.796

1.84

Answer:

the order would go 1.7, 1.749, 1.75, 1.775, 1.796, then 1.84.

Step-by-step explanation:

becuase if you ignore how long the numbers are, and focus on numerical order like 1, 2, 3, and apply that to this, this becomes the order.

I am thinking of a number. The sum of its digits is divisible by 2. My number is a multiple of 11. My number is greater than 4 x 5 but less than 7 x 8+ 23. The number is a multiple of 3. What is my number? Is it possible to have more than one answer? ​

Answers

Answer:

66

Step-by-step explanation:

Number:xy

20<xy<79

Multiples of 2,3,11

The least common multiple of this number is 66.

In the figure below, triangle ABC undergoes a reflection and a translation to become triangle PQR. In triangle ABC, m

The angle measures of triangle PQR are: m

Answers

Answer:

Step-by-step explanation:

A corresponds to angle P, so angle P measues 70 degrees.

Similarly, angle Q measures 60 degrees and angle R measures 50 degrees.

helpppp :) 50 points
An equation is shown below:

4(2x – 5) = 4

Part A: How many solutions does this equation have? (4 points)

Part B: What are the solutions to this equation? Show your work. (6 points)

Answers

Answer:

x = 3

Step-by-step explanation:

A only 1 solution

B

4(2x - 5) = 4 ( divide both sides by 4 )

2x - 5 = 1 ( add 5 to both sides )

2x = 6 ( divide both sides by 2 )

x = 3

Answer:

[tex]\boxed{\sf{x=3}}[/tex]

Step-by-step explanation:

Isolate it by the term of x from one side of the equation.

Part A. There will be one solution.Part B.

4(2x-5)=4

First, divide by 4 from both sides.

[tex]\sf{\dfrac{4\left(2x-5\right)}{4}=\dfrac{4}{4}}[/tex]

Then, solve.

4/4=1

Rewrite the problem down.

2x-5=1

Add by 5 from both sides.

[tex]\sf{2x-5+5=1+5}[/tex]

Solve.

1+5=6

2x=6

Divide by 2 from both sides.

2x/2=6/2

Solve.

Divide the numbers from left to right.

6/2=3

x=3

Therefore, the correct answer is x=3.

I hope this helps you! Let me know if my answer is wrong or not.

Help is number 2 correct and help for number one​

Answers

Number 2 is incorrect :

[tex]\frac{-25x^{3}+15x }{5x} = \frac{-25x^{3}}{5x} + \frac{15x}{5x} = -5x^{2} + 3[/tex]

Number 1 :

[tex](-5)^{3} . 13^{-2} . (-5) .13^{9} = -125 . \frac{1}{13^{2}} .(-5).13^{9} = 625 . \frac{13^{9}}{13^{2}} = 625 . 13^{7}[/tex] (i dont think we can bare calculate this cause it;s too big ;-;)

The height of the prism is the Blank between the bases.

Answers

Answer:

so how do i find the earea

Step-by-step explanation:

graph "-3n+4<25" on a line plot

Answers

Answer:The solution is in the attached file

Step-by-step explanation:


Given the equation of a circle. X^2+ y^2 - 6x + 2y +1 = 0, find the coordinates of the center and the length of the radius.
Coordinates for the center=
Length of the radius =

Answers

Answer:

I know the answer but you may select me as Brainliest

Answer:
Center: (3, -1)
Radius: 3

Steps:
Rewrite in standard form to find the center
(h,k) and radius.

x^2 + y^2 - 6x + 2y + 1 = 0
Standard Form: (x - 3)^2 + (y + 1)^2 = 9

Center: (3, -1)
Radius: 3

Tiana bought 3 video games that each cost
the same amount. The tax on each video
game was $1.29. She paid a total of $68.37
after tax. What was the price of each video
game, before tax?
a

Answers

well, we know she paid a total after tax of $68.37, we also know that each game had a tax of $1.29, so 3 games that'll be 3 * 1.29 = 3.87.

so if we take out the tax from 68.37, that'll be 68.37 - 3.87 = 65 bucks.

we know each game cost the same amount so 65 ÷ 3 ≈ 21.67 before tax.

Tiana bought 3 video games at a cost of $22.36 each, before tax. The total amount paid after tax was $68.37.

Given that,

Tiana bought 3 video games.

The cost of each video game was the same.

The tax on each video game was $1.29.

Tiana paid a total of $68.37 after tax.

To find the price of each video game before tax,

Start by subtracting the total tax from the total amount paid.

Tiana paid a total of $68.37 after tax and the tax on each video game was $1.29.

So, subtract the tax from the total amount to get the price before tax.

$68.37 - $1.29 = $67.08.

Since Tiana bought 3 video games,

Divide the total price before tax by 3 to find the price of each video game.

$67.08 ÷ 3 = $22.36.

Hence, the price of each video game before tax was $22.36.

To learn more about subtraction visit:

https://brainly.com/question/17301989

#SPJ3

What are the y-intercept and the horizontal asymptote of g(x) = 4x + 3?
(0, 3); y = 4
(0, 4) ; y = 3
(0, 5) ; y = 4
(0, 7) ; y = 3

Answers

Using exponential function concepts, it is found that that the y-intercept and the horizontal asymptote are given by, respectively:

(0, 3); y = 4

What is an exponential function?

It is modeled by:

[tex]y = ab^x + c[/tex].

In which:

a is the initial value.b is the rate of change.y = c is the horizontal asymptote.

In this problem, the equation is:

[tex]y = 4^x + 3[/tex].

Hence the horizontal asymptote is y = 3, and the y-intercept is y when x = 0, hence:

[tex](0,y) = (0, 4^0 + 3) = (0, 4)[/tex]

Which means that the first option is correct.

More can be learned about exponential function concepts at https://brainly.com/question/25537936

Answer:

B. (0, 4) ; y = 3

Step-by-step explanation:

I took the exam for algebra 2

The rule for the number of fish in a home aquarium is 1 gallon of water for each inch of fish length. Marta's aquarium holds 39 gallons of water and Hank's aquarium hold 51 gallons of water. The aquarium holds two types of fish, fish A and fish B. If Marta bought 3 of fish A and 2 of fish B, and Hank bought 3 of fish A and 4 of fish B, how long is fish A and how long is fish B?

Answers

Answer: Fish A: 9 inches ; Fish B: 6 inches

Step-by-step explanation:

This can be turned into a systems of equation, where x equals the length of fish A and y equals the length of fish B:

3x + 2y = 39 → -3x - 2y = -39

3x + 4y = 51 → 3x +4y = 51

Combine: 0 + 2y = 12 → y = 6 inches.

Plug back in: 3x + 2(6) = 39 → 3x = 27 → x = 9 inches.

Four eighths of kelly’s beads are red, 18 are white, and the rest are blue.
what fraction of kelly’s beads are blue?

Answers

4/8=1/2 so Kelly blue beads are also 4/8. Sorry if I’m wrong.

Two rectangles share a common side as shown.
Determine whether each expression can be used to determine the
total area, in square feet, of the two combined rectangles.
ft
Select Yes or No for each expression.
7.52 + 20
o Yes
Ο Νο
*
7.5 ft
x ft
27.5 + 2
Yes
Ο Νο
20 (7.5 + x)
o Yes
Ο Νο
150 + 3
Yes
Ο Νο
202 + 150
O Yes
No

Answers

Answer:

yes they are

Step-by-step explanation:

A triangular pyramid has a base shaped like an equilateral triangle. The legs of the equilateral triangle are all 1 yard long, and the height of the equilateral triangle is 0.9 yards. The pyramid's slant height is 1 yard. What is its surface area?

Answers

Answer:

base: 1/4 * 12^2 * √3 = 36√3

each face: 1/2 * 12 * 8 = 48

Now finish it off

Step-by-step explanations:

Pls help dont need an explanation just an answer

Answers

At y-intercept x=0.
Y-intercept = 1

(0,1). & (-1 ,-4)

Slope =y2-y1/(x2-c1)
=(-4-1)/(-1-0)
=-5/-1
Slope =5
= (

Answer:

[tex]\boxed{\text{Slope = 5}}[/tex]

[tex]\boxed{\text{y-intercept = 1}}[/tex]

Step-by-step explanation:

To find the slope of the line, we must pick any two points on the line. Using those two points, we will calculate the Rise and the Run. Then, we will use the slope formula (Rise/Run) to find the slope. The y-intercept is the intersection of the point on the y-axis.

[tex]\text{My chosen points: (0,1) and (1,6)}[/tex]

[tex]\text{Rise = 5; Run = 1}[/tex]

≡≡≡≡≡≡≡≡≡≡≡≡≡≡≡≡≡≡≡≡≡≡≡≡≡≡≡≡≡≡≡≡≡≡≡≡≡≡≡≡≡≡≡≡≡≡≡≡≡≡≡≡≡

[tex]\text{Slope}= \dfrac{\text{Rise}}{\text{Run}}[/tex]

➡ [tex]\bold{Slope = \frac{5}{1} = \underline{5}} \ \ \ \ \ \ \ \ \ \ \ \ \ \ \ \ \ \ \ \ \ \ \ \ \ \ \ \ \ \ \ \ \ \ \ \ \ \ \ \ \ \ \ \ \ \ \ \ \ \ [\text{Rise = 5; Run = 1}][/tex]

[tex]\underline{........................................................................................................................}[/tex]

[tex]\text{Intersection on y-axis:} \ (0,1)[/tex]

➡ [tex]\text{y-intercept} \rightarrow (0,\bold{ 1}) \rightarrow \bold{\underline{{1}}}}[/tex]

≡≡≡≡≡≡≡≡≡≡≡≡≡≡≡≡≡≡≡≡≡≡≡≡≡≡≡≡≡≡≡≡≡≡≡≡≡≡≡≡≡≡≡≡≡≡≡≡≡≡≡≡≡

Pls help me
AND

PLS FOLLOW THE RULES

TROLL FOR POINTS= REPORT

FIRST AND CORRECT ANSWER= BRAINLIEST

( NO LINKS ) at all

Answers

Pythagorean Theorem: a^2 + b^2 = c^2

a = 23 m

b = 11 m

c^2 = (23 m)^2 + (11 m)^2 = 650 m^2

c = sqrt(650 m^2) = 25.5 (rounded to nearest tenth)

Jill bought 3 pints of blueberries for $3.02 each. What is the best estimate of how much Jill spent

Answers

Answer: 9.06

Step-by-step explanation:

Do; 3.02+3.02+3.02

This adds up to 9.06


Fifteen less than the square of a number is the same
as twice the number. Find the number.

Answers

Call the number : a

We have : [tex]a^{2} - 15 = 2a[/tex]
[tex]- > a^{2} - 2a = 15 - > a(a-2) = 15[/tex]

Now, we list all the numbers that can be multiplied to get 15 :

1 x 15 (and reverse)
3 x 5 (and reverse)

-1 x (-15) (reverse)
-3 x (-5) (a l s o r e v e r s e)

We substitute in :

If a = 1 -> a(a-2) = 1 x (-1) = -1 (not equal 15)
If a = 3 -> a(a-2) = 3 x 1 = 3 (not equal 15)
If a = -1 -> a(a-2) = -1 x (-3) = -3 (also not equal 15)
And if a = -3 -> a(a-2) = -3 x (-5) = 15 (satisfied)

So, the number is -3

Recheck : -3(squared) - 15 = 9 - 15 = -6
Twice of -3 = -6

Hello!

[tex]========================================[/tex]

First of all, let the unknown number be z.

Then, "the square of z" means "z squared" which is the same as "z times z" or just z².

Now, "fifteen less than the square of z" means we subtract 15 from z²:

z²-15

Then, this equals twice the number, or 2 times z:

z²-15=2z

[tex]========================================[/tex]

Notes:-Hope everything is clear.Let me know if you have any questions!Always remember: [tex]\boxed{Knowledge~is~power!}[/tex]Enjoy your day!Answered by[tex]\boxed{An~Emotional~Helper}[/tex]

:-)

Find the value of log 1 2 462 to four decimal places. –8.8517 –0.1130 0.1130 8.8517

Answers

The logarithm value of log 1/2 462 to four decimal places is  –8.8517.

What is a logarithm?

The logarithm is the mathematical inverse function to exponentiation.

The logarithm of a number x is the exponent to another fixed term.

It is given the log of 462 to a base of 1/2.

[tex]log_{0.5} 462 = \frac{log462}{log0.5}[/tex]

 = –8.8517

Therefore, the logarithm value of log 1/2 462 to four decimal places is  

–8.8517.

Learn more about logarithm :

https://brainly.com/question/3181916

Other Questions
Mathematics course 3 common core 2013 edition solutions A sample of PH3, occupying 1.50 L at 25 oC and 0.80 atm, reacts with 0.080 atm, reacts with 2.28L of O2 at 20.0 oC and 0.100 atm according to the following equation: 4 PH3(g) + 8 O2(g) => P4O10(s) + 6 H2O(g) How many grams of each product is produced? How many grams of each reactant are left at the end of the reaction? What is the final pressure at the end of the reaction? (The final volume is 3.88L and the final temperature is 22.5C). Brainliest if correct The following data relate to the operations of Shilow Company, a wholesale distributor of consumer goods: Current assets as of March 31: Cash $ 8,800 Accounts receivable $ 25,200 Inventory $ 47,400 Building and equipment, net $ 114,000 Accounts payable $ 28,425 Common stock $ 150,000 Retained earnings $ 16,975 The gross margin is 25% of sales. Actual and budgeted sales data: March (actual) $ 63,000 April $ 79,000 May $ 84,000 June $ 109,000 July $ 60,000 Sales are 60% for cash and 40% on credit. Credit sales are collected in the month following sale. The accounts receivable at March 31 are a result of March credit sales. Each months ending inventory should equal 80% of the following months budgeted cost of goods sold. One-half of a months inventory purchases is paid for in the month of purchase; the other half is paid for in the following month. The accounts payable at March 31 are the result of March purchases of inventory. Monthly expenses are as follows: commissions, 12% of sales; rent, $3,600 per month; other expenses (excluding depreciation), 6% of sales. Assume that these expenses are paid monthly. Depreciation is $855 per month (includes depreciation on new assets). Equipment costing $2,800 will be purchased for cash in April. Management would like to maintain a minimum cash balance of at least $4,000 at the end of each month. The company has an agreement with a local bank that allows the company to borrow in increments of $1,000 at the beginning of each month, up to a total loan balance of $20,000. The interest rate on these loans is 1% per month and for simplicity we will assume that interest is not compounded. The company would, as far as it is able, repay the loan plus accumulated interest at the end of the quarter. Required: Using the preceding data: 1. Complete the schedule of expected cash collections. 2. Complete the merchandise purchases budget and the schedule of expected cash disbursements for merchandise purchases. 3. Complete the cash budget. 4. Prepare an absorption costing i A grocer wants to mix two kinds of candy. One kind sells for $1.15 per pound, and the other sells for$2.75 per pound. He wants to mix a total of 24 pounds and sell it for $1.30 per pound. How many pounds of each kind should he use in the new mix? (Round off the answers to the nearest hundredth.) chinua reacts 37g of magnesium with 150g of sulfuric acid. what will be the total mass of the products of this reaction Karen's mom is filling 12 birthday hats with candy for her birthday party. Each hat is shaped like a cone with a height of 6 inches and a radius of 3 inches.Which statements are true? Choose all that are correct. Tell me about Harriet Tubman. What was her nickname? Why did she get this nickname?Where was she from? What did she do? How was she treated by those in the South? How wasshe treated by those in the North? I need the answer to this quick pls Which actions could be categorized in the both section of the Venn diagram?Check all that apply.requires oxygenstarts with glycolysisoccurs in the mitochondrionproduces ATP Fast Food Nation5. Page 201: "A government health official . . . compared the sanitary conditions in a modern feedlot with those in a crowded European city during the Middle Ages, when people dumped their chamber pots out the window, raw sewage ran in the streets, and epidemics raged." What is the point of making this comparison between conditions in cattle feedlots in the 21st century and the living conditions in crowded cities during the Middle Ages? Darin has clicked on the Outdoor Gear website. What should the marketing manager have already done to make sure the site works for Darin How are some autoimmune disorder symptoms alleviated by using medications that suppress the immune system Read the following text from a student essay. How can the writer best improve his or her sentence fluency?Eva Benton is my best friend for many important reasons. She is a humorous person. I laugh with her all the time. In addition, Eva is a good listener. I trust her completely. I can confide in her about anything. She gives me good advice, too. Eva is so much fun to be with. We always have a good time together. I hope Eva will be my best friend for life.by using shorter sentencesby using varied sentence structures What is your opinion of this statement?Limits are only limits if you accept them as such.AgreeDisagreeExplain why you voted the way you did. Which statement correctly describes a feature of the rock cycle?O Rocks are preserved through the process.O Rocks change from one type to anotherDifferent rock groups are not related to one another.The rock cycle follows a specific order.Mark this and retumSave and ExitNextSubmit what divided by 4 is 8? What is the theorem that states a perpendicular line to the hypotenuse splits the triangle into two similar right triangles? I forgot and I need this for statement-reason proof. an answer would be appreciated Last week, Harry drank too much tequila and this made him vomit. Now, just the smell of tequila makes his stomach a bit queasy. Vomiting is an example of a(n): Australia is the worlds leading producer of __________.A.diamondsB.goldC.bauxiteD.iron ore